Contracts 4% Quizes

Pataasin ang iyong marka sa homework at exams ngayon gamit ang Quizwiz!

A man owns a small apartment building in town. Several of his tenants have recently moved out, so the owner hires a painter to paint one of the apartments while it is vacant. The painter agrees to paint the vacant apartment for $1,200, and she is to complete the work within 30 days. The parties sign a contract stating these terms. Later that day, as the apartment owner is reviewing his calendar, he realizes that new tenants will be moving into the apartment that the painter is going to repaint in 15 days, not 30 days. The owner calls and asks the painter if she can complete the job within 14 days, rather than 30 days. The painter agrees, but only if the owner promises to pay her an additional $300. The owner reluctantly agrees to the new amount. The painter paints the apartment within seven days, but the owner refuses to pay her the additional $300 and remits payment of only $1200. If the painter files suit against the apartment owner to recover the additional $300, who is more likely to prevail? A. The painter because she made a new promise to the apartment owner in exchange for the higher payment. B . The apartment owner because the painter tried to extort the additional money knowing that the owner was under duress. C. The apartment owner, because his request to paint the apartment in 14 days, rather than 30 days, was reasonable. D. The apartment owner, because the painter had a pre-existing duty to paint the apartment.

A is the best answer. This questions tests the concept of preexisting legal duty. The R2K explains in Sec. 73 that performance of a legal duty already owed is not consideration. If we do not have consideration, we cannot have a contract. In this example, the painter already had a contractual duty to paint the apartment. However, the painter made a new promise when she promised to paint the apartment in 15 days (rather than 30 days). That new promise is consideration. The preexisting legal duty rules prevents one-sided contract modifications, but in this hypo, we do not have a one-sided modification because both parties make a new promise. The painter promises to finish the job faster and the apartment owner promises additional compensation. B is not the best answer because although the apartment owner was in a tough spot, the painter is not obliged to gratuitously modify her contract duties because the owner misread his calendar. This is not a situation like Alaska Packers. C is not the best answer. A one-sided modification might be enforceable if there are unanticipated changed circumstances and the modification is fair and equitable (or fair and reasonable) in light of those circumstances, that doctrine does not apply here. In this case, both parties agreed to a modification. D is not the best answer. Although the painter had a pre-existing contractual duty to paint the apartment, she did not have a duty to do so in 15 days.

Mel's Hardware sells portable generators for $400 each. Mel's agrees to sell a generator to Pop for $400. Before Pop comes in to pick it up, a vicious blizzard hits the area, dropping power lines and cutting off electricity for hundreds of square miles. Mel's is flooded with people who want to buy generators. Mel's triples the price to $1,200 each and sells several of them. When Pop comes in to pick up his generator, he demands the generator but Mel's refuses to sell it unless he pays an additional $800. Pop refuses. Mel's explains that it has a line of people already waiting to buy the generator at the higher price. Pop agrees to pay the higher price and Mel's hands over the generator. If Pop sues to get his $800 back, his best argument will be... A. A. The contract modification was entered into as a result of fraud. B. The contract was entered into as a result of misrepresentation. C. The modification was not fair and equitable and was not made in good faith. D. The contract was entered into as a result of economic duress.

Although fraud can be a defense to contract, A is not the best answer because there is no fact in the fact pattern to indicate fraud. Although misrepresentation can be a defense to contract, B is not the best answer because there is no fact in the fact pattern to indicate misrepresentation or fraud. C IS THE BEST ANSWER. Modified contracts are really new contracts, so generally each side must have consideration (and also offer & acceptance). We refer to this as the pre-existing legal duty rule. The modification in this example is not legally enforceable because only Pop's contractual duties were modified. There is an exception to the pre-existing legal duty rule that permits one-sided modifications where an unanticipated change in circumstance makes a one-sided change fair and equitable in light of changed circumstances. The facts in this question do not support application of the exception. Mel's performance (handing over the generator) has not become more difficult due to the increased demand -- it has just become less attractive. Compare this question to the facts of Alaska Packers. D is not the best answer. Although duress can be a defense to contract, the facts do not support a claim of economic duress here. Economic duress occurs when an improper threat puts economic pressure on a party to enter into an agreement. Here, Mel's desire to make more profit does not support an argument that Mel suffered economic duress.

A female horse won the most competitive horse race in the United States three years ago. Since her retirement, her owner has not been able to successfully breed her. A veterinarian specializing in new techniques for breeding animals that have not been able to conceive approaches the horse owner, explaining that the veterinarian has always been a devotee of horse racing and has had a lifetime ambition to own a winning horse. These facts are true, but the veterinarian does not tell the owner that the veterinarian wishes to try the new technique on the female horse. The owner sells the female horse to the veterinarian for $50,000, a generous price for a sterile, retired racehorse, but far less than the worth of the horse if she can bear offspring. Soon after the veterinarian purchases the female horse, the veterinarian extracts eggs from the horse's ovary, fertilizes them in a test tube, and implants the fertilized egg in the horse's womb. When the owner learns that the female horse is pregnant and the sire is another winner of the most competitive horse race in the United States, the owner sues the veterinarian to rescind the sale. Assume for the purposes of this question that assisted reproduction is permitted in the thoroughbred industry. Which party is likely to prevail in this action? A. The veterinarian will prevail, because the owner has no legal basis on which to rescind this valid contract. B. The owner will prevail, because his ignorance of the fact that female horse was capable of successfully being bred constitutes a unilateral mistake. C. The owner will prevail, because a mutual mistake is always grounds for rescinding a contract. D. The owner will prevail, because the veterinarian failed to disclose that the veterinarian specialized in the breeding of previously infertile animals.

Answer choice A is the best answer for this challenging question. The veterinarian does seem to have withheld important information, which in the right circumstances, can result in a colorable defense for the victim (here the owner). Actionable non-disclosure, however, requires that a putative victim first establish that the non-disclosing party had some duty to speak or disclose the information. There are a few ways that duty can be established. Of those triggers, two are worth taking a hard look at (1) intentional concealment and (2) correcting a mistake. Intentional concealment is "the act of the cover-up." While the vet didn't share information about their specialty in breeding, we do not have any evidence that the veterinarian actively covered up a material fact. It's possible that the vet's practice area is "readily observable" by, for example, checking out the vet's LinkedIn profile or practice website. Similarly, no fact supports active concealment (e.g., painting over a defect) or indirect concealment (e.g., preventing the other party from making an investigation). Choice B is not the best answer because it is not clear that the fact that the horse was incapable of breeding was a mistake. In fact, the horse had been incapable of breeding up to this point. The vet purchases the horse hoping the vet's techniques will result in a horse capable of breeding, but not knowing for sure. Choice C is not the best answer choice because is contains a misstatement of law. Mutual mistake is not *always* grounds for rescission. Choice D is perhaps the second-best choice, but for reasons explained above, the vet did not have a duty to disclose the vet's specialization.

Which of the following is NOT a basis for the termination of an offer: A. Death or incapacity of the offeror. B. Fraud, misrepresentation, duress, or other similar defect in the negotiation process. C. Revocation. D. Rejection.

Fraud, misrepresentation, duress, or other similar defect in the negotiation process might render a contract unenforceable, but does not mean an offer is terminated.

Colette Contractor enters into a contract with Helena Homeowner to install an in-ground swimming pool, including a complicated water slide, in Helena's backyard. Helena lives in East Dakota, a jurisdiction that follows the majority rules. The contract provides that the cost of the materials will be $35,000 and the labor will be $18,500. The contract specifies that "all installation services will be done in a professional and workmanlike fashion." It also provides that Colette is not making any warranties about "any materials or products installed. Any warranties for materials or products are provided by the manufacturer only." What body of law applies to this transaction? A. Definitely the Common law, because Colette is a service provider. B. Definitely the UCC Article 2, because the value of the goods far exceeds the value of the services. C. Probably the CISG, because the CISG constitutes federal law that supplants inconsistent state law. D. Probably the UCC. Applying the majority rule--the predominate purpose test--this transaction is predominately about goods therefore the UCC applies.

The best answer is D. It's not always true because very little is always true, but you should be very skeptical of any law school answer that starts with "definitely." Law is intrinsically uncertain and contingent. It depends on arguments and predictions about what a decision maker will conclude. Both A and B are far, far too "certain" to be good answers. That's especially true when there's an accurate and far more balanced option on the board. D is correct, at least about the test that will be applied. And, using that test, there are factors that tilt both ways. The value of the goods versus the services tilts pretty strongly towards this being a UCC transaction. But the nature of a contractor's business is probably mostly about services. And the language of the contract is sort unclear about whether this is a services or a goods transaction. The references to professional and workmanlike installation, tilt towards this being a services deal. The references to warranties could cut either way, I suppose, but the fact that the contractor is saying she's not making any product or material warranties suggests that she's not seeing herself as primarily a seller of goods. That probably tilts towards this being a service. Ultimately, this is a tough call. I'm not sure that I agree with the first part of D, that it is probably a transaction governed by the UCC, but the contingent nature of this answer -- the fact that it recognizes that there are competing arguments and that this will be a close call -- makes it significantly better than either A or B. And C is just wrong. It's also something that I promised not to test you on -- the CISG--so I would not put this as a correct answer on the final.

A car dealership places an advertisement in the local newspaper, stating that it will sell a "like new" Suburu Outback (normally between $25,000 and $30,000) for "$.19000.00" to the "first five customers to come to the dealership on Wednesday." The first customer to arrive at the dealership tenders his $.19 to the salesperson, who refuses to sell him the car. The dealership tells the customer that the price in the circular was a misprint. May the customer enforce a contract to sell the sedan car for the advertised price? A. Yes, because the advertisement was an offer, and the customer unequivocally accepted the offer when he tendered his money. B. Yes, because the "first five customers" language made the advertisement an offer rather than an invitation for offers. C. No, because the advertisement was not an offer but rather merely an invitation to make offers. D. No, because no reasonable person would believe that the offer to sell the sedan car for "$.19000.00" was anything other than a misprint.

The best answer is: (D) No, because no reasonable person would believe that the offer to sell the car for "$.19000.00" was anything other than a misprint. The advertisement came much closer to being an offer when the store included the "first five customers" language. Communication of an offer to multiple offerees is generally an invitation for an offer rather than an offer unless that communication indicates how goods will be allocated in the event of excess demand, such as by limiting the offer to "first come, first served" or a certain number of customers. However, an offer that is too good to be true, and which a reasonable person would not believe to be a valid offer, is not an offer. In this case, because no reasonable person would believe that the dealership would sell sedan cars for 19 cents, particularly by using an advertisement that stated the price in such a nonsensical way, as "$.19000.00," this language does not constitute an offer. (A) Incorrect. Yes, because the advertisement was an offer, and the customer unequivocally accepted the offer when he tendered his money. This answer misses the one crucial fact that makes this language not an offer: the obvious misprint that means no reasonable person would believe it really was an offer. Generally, it is true that advertising can be treated as an offer, if as here language such as "first five customers" is included, and it is also true that the customer in this case communicated his acceptance. However, offers that a reasonable person would consider to be "too good to be true" are not offers. In this case, the 19-cent price was "too good to be true" and simply the obvious misplacement of a decimal. Therefore, the advertisement did not constitute an offer that could be accepted. (B) Incorrect. Yes, because the "first five customers" language made the advertisement an offer rather than an invitation for offers. While, generally, communication of an offer to multiple offerees is a valid offer if it specifies how goods will be allocated if there is excess demand, if no reasonable person would interpret the communication as an offer because it is too good to be true, there is no offer. In this case, the language that otherwise would be an offer--the price of the sedan plus the statement that the first five customers will get the stated price--loses the status of an offer because no reasonable person would believe a sedan car would sell for 19 cents, particularly in light of the obvious typographical error. (C) Incorrect. No, because the advertisement was not an offer but rather merely an invitation to make offers. This answer is incorrect because the problem is not that the alleged "offer" was in an advertisement. Although it is generally true that an advertisement is not an offer, but instead is an invitation to make offers, here the advertisement included the language "first five customers." Such communication, which specifies how goods will be allocated when there is excess demand, constitutes an offer. However, if a reasonable person would view the offer as too good to be true, it is not a valid offer.

Two acquaintances, Link and Zelda, were drinking at a bar in Uptown. Link had always admired Zelda's car, and in fact Link had offered to purchase Zelda's car in the past, but Zelda had always declined to sell it. After several drinks, and a lengthy discussion of how great Zelda's car was, Link said, "You know I'll give you $50,000 for your car." Zelda scoffed, "You don't have $50,000!" Link protested that she could put the money together within two days, and wrote out the following on the bar napkin, "I'll pay you $50k for your car. /s/ Link." Zelda shrugged, and took the napkin and stuck it in her pocket. Link's face broke into a big grin and Link said, "It's a deal, then!" If Zelda does not want to follow through by selling her car to Link, what is Zelda's best argument that there is not a contract? A. Since the parties were both drinking, there was no mutual assent. B. Since the napkin was signed by only one party, there cannot be a contract. C. Zelda did not intend to sell her car to Link, and under the objective theory of contract formation, simply shrugging is insufficient to manifest assent to the exchange. D. Zelda did not ever intend to sell her car to Link, therefore, under the subjective theory of contract formation, Zelda's intent not to enter into the agreement is sufficient to prevent a contract from forming.

A is not the best answer because although it is possible to be too drunk to be competent to contract, the facts do not indicate such excess, and the answer choice suggests that drinking obviates mutual assent, which is an incorrect statement of law. Answer B is not a correct statement of law. The Statutes of Frauds requires a writing signed by the party to be charged, not by both parties. D is incorrect because we do not follow a subjective theory of contract formation. That leaves C as the best answer.

Professor Butterfoss has a foosball table in his basement. Professor Holcomb has long coveted that foosball table, and she has offered to buy it on more than one occasion. Now that Prof. Butterfoss's children are all grown and out of the house, he does not use the foosball table very often. Butterfoss likely could sell the foosball table for around $100 on Craigslist or a similar resale site. Butterfoss texts Holcomb, "Still want that foosball table? If you do, come on over and pick it up whenever. You might need somebody to help you carry it. It's heavy." Holcomb and her strongest friend show up at Butterfoss's house to fetch the table. What is the best explanation of the parties rights and obligations at this point? A. There is likely a contract between Holcomb and Butterfoss because the promise of the foosball table induced Holcomb to incur the detriment of getting a friend and driving over to pick up the foosball table. B. Butterfoss made an offer that Holcomb can accept by performance -- in other words, this is a unilateral contract that will be formed after Holcomb picks up the foosball table. C. There is no contract because Holcomb didn't promise to pay anything so there is inadequacy of consideration. D. There is no contract between the parties because Butterfoss promise to give Holcomb the foosball table is most likely a promise to make a gift.

A is not the best answer. While it is true that Holcomb can point to a detriment, that is only half the test. On these facts, there is not good evidence that Butterfoss was motivated to induce Holcomb to pick up the foosball table when he said she could have it. This looks more like a conditional gift. B is not the best answer. There is not likely to be a contract here -- whether that contract is unilateral or bilateral. While it is true that Holcomb can point to a detriment, that is only half the test. On these facts, there is not good evidence that Butterfoss was motivated to induce Holcomb to pick up the foosball table when he said she could take it. This looks more like a conditional gift. C is not the best answer. Generally, courts do not inquire into adequacy of consideration so long as there is sufficient consideration. Consideration does not have to be money. In this example, if Prof. Butterfoss's promise to give Holcomb the foosball table was induced by Holcomb's promise to pick up the foosball table (or Holcomb actually picking up the foosball table) there would be consideration. You can imagine that getting rid of an unwanted piece of heavy furniture is worth something to Butterfoss. The best answer is D. While it is possible to have a bargained for exchange of a foosball table for picking up that table, the facts of the problem do not support an argument that Holcomb's removal of the foosball table induced Butterfoss to make the promise to give the table to Holcomb. The line between conditional gift and bargained for exchange (consideration) can be difficult to draw and is fact-specific.

Following a particularly heavy snow, Holcomb texts the teenager who lives across the street the following, "I am leaving for work and will be teaching all day. I won't have my phone. I promise to pay you $15 if you shovel my front walk before I get home from work." Which of the following statements best describes the parties obligations? A. This is a unilateral contract, and the contract forms only after performance has been completed. B. Since the teenager neighbor will not be able to communicate her promise to shovel to Holcomb, there will be no contract. C. A contract is formed as soon as the teenage neighbor begins shoveling. D. There is no contract because the teenager neighbor did not have a chance to bargain, and consideration requires bargained for exchange.

A is the best answer. Our textbook uses the terms "bilateral contract" and "unilateral contract." See page 118 for the textbook definitions. (Note that the R2K has moved away from those terms, but has not abandoned the conceptual distinctions.) Just as an exchange of promises can create a contract (assuming the requirements are met) an exchange of a promise for performance can also support a contract.

A dog owner owned a rescue dog of great sentimental, but little monetary, value. One day, the dog disappeared, and the owner placed the following advertisement in the newspaper: "I will pay $500 to anyone who returns my dog." A man found the dog the next day. Knowing to whom the dog belonged, the man took the dog home to the man's own home, intending to return it the next day. Before he returned the dog, the man read the dog owner's advertisement in the newspaper. Assuming that the dog owner's advertisement was an offer, what are the rights of the man who found the dog? A. He can likely recover $500, as he has accepted the offer. B. He can likely recover $500 if he returns the dog to the owner. C. He likely cannot recover the $500, even if he returns the dog, because he did not have knowledge of the offer when he found the dog. D. He cannot likely recover the $500 because this is not the sort of promise that the law will enforce.

B is the best answer. Most of the contracts we will study involve bilateral contracts (a promise for a promise). A unilateral contract involves a promise in exchange for performance. The most straightforward unilateral contract is probably the reward example. Assuming that the ad was an offer, as the problem instructs, it was an offer for a unilateral contract which would be accepted and completed when the man completes the requested performance -- returning the dog.

Attorney offered Friend, who is also an attorney, an opportunity to go into business with her. Attorney said, "I will give you a 50% stake in my law firm if you give me $50,000. This offer will remain open for two weeks." One week later, Friend rejected the offer. Two days after that, Friend had a change of heart and called Attorney. Friend said, "I changed my mind. I accept your offer." Attorney said nothing. Have Attorney and Friend formed a contract? A. Yes. Attorney accepted Friend's new offer by Attorney's silence. B. Yes. The parties did form a contract because Friend accepted the offer well before the offer was scheduled to lapse (in two weeks). C. No. The parties did not form a contract because Friend made a counter-offer that attorney did not accept. D. No. The parties did not form a contract because Friend rejected Attorney's offer, and Attorney did not accept Friend's offer.

D is correct. An offeree's power of acceptance is terminated by his rejection of the offer. That would be true even if the offeror had given up the power to revoke the offer by granting the offeree an option. But, in this case, there's no evidence that an option even exists. Remember, an option contract is a little mini contract. The offeror promises not to revoke in exchange for some sort of consideration from the offeree. Here, we have no consideration from the offeree. Options must be in writing, and this agreement was oral. A is not the best answer. Silence is almost never sufficient to be acceptance. B is not the best answer. Friend's power of acceptance was terminated when he rejected the offer. C is not the best answer. Friend's power of acceptance was terminated immediately once he rejected the offer. The question does not give facts that suggest there was a counteroffer.

Two acquaintances, Link and Zelda, were drinking at a bar in Uptown. Link had always admired Zelda's car, and in fact Link had offered to purchase Zelda's car in the past, but Zelda had always declined to sell it. After several drinks, and a lengthy discussion of how great Zelda's car was, Link said, "You know I'll give you $50,000 for your car." Zelda scoffed, "You don't have $50,000!" Link protested that she could put the money together within two days, and wrote out the following on the bar napkin, "I'll pay you $50k for your car. /s/ Link." Zelda takes the napkin, signs her name to it as well and shakes hands with Link. Zelda says, "You have a deal!" At that moment, Link comes to her senses and realizes she cannot put the money together. Link shouts, "No deal! Never mind! Forget it!" A. Link's rescission will be valid since it was almost contemporaneous with Zelda's acceptance. B. Link's rescission will be valid since she has three days to change her mind about the purchase of a used car. C. Link's recission will be valid since Zelda has not yet relied to Zelda's detriment on the contract. D. Link's recission was not valid.

D is the best answer. Once a contract is formed, which happened when Zelda signed her acceptance, both parties are bound. Some jurisdictions have rescission periods for certain types of contracts, but the problem does not suggest we are in such a jurisdiction, so B is not the best answer. Detrimental reliance is also not required to enforce a contract, so C is not the best answer. If you answered A, stop it.

Grant really wants to star in Olivia's newest local commercial for a car dealership. He has aspirations to one day make it as a real celebrity. On August 1, Grant receives an email from Olivia offering him a co-staring role in the commercial, providing details about the shoot and the compensation. She says that the commercial will shoot on August 5, so she needs a quick response. The same day he received the email, Grant writes Olivia a letter accepting her offer and drops it in the mail. Meanwhile, Oliva has a change of heart and decides that she doesn't have a sufficient budget to hire Grant. On August 2, not having heard from Grant yet, she sends him another email revoking her offer. Grant receives this second email at the end of the day on August 2. Olivia receives Grant's letter purporting to accept her original offer on August 3. Grant now sues Olivia for breach of contract. What is Olivia's best defense? A. Olivia is master of the offer and rejected before Grant accepted. B. Oliva's offer lapsed because it was not accepted within a reasonable period of time. C. Olivia made an offer for a unilateral contract, so Grant could only accept by full performance, which he did not render. D. Oliva revoked her offer before it was accepted because applying the mailbox rule in this situation would be unreasonable.

D is the best option on the board for this challenging question. The mailbox rule or the "deposited mail rule" provides that acceptance may be effective when the acceptance leaves the control of the offeree so long as (a) the mode of acceptance is reasonable under the circumstances and reasonably consistent with the offeror's expectations, (b) the acceptance is properly addressed to the offeror and the offeree reasonably believes that it will reach the offeror, and (c) the acceptance is not for an option contract. Here, requirement (a) may well be violated. Olivia emails her offer to Grant and specifically indicates that she needs a quick response. Grant decides, despite both the mode of delivery of her offer and her express imposition of a timely response, to accept through snail mail. This might not be reasonably consistent with the offeror's expectations and thus might undercut the effectiveness of the acceptance through the deposited mail rule. A is not the best answer. Although Olivia is the master of the offer, offerors do not "reject." B is tricky. It's not a bad answer, but it isn't the best option available because it overlooks Grant's likely counterargument -- that he accepted via deposited mail. Moreover, because the offer was made via email, even though it says that Olivia needs a "quick" response, there's no evidence to suggest that the offer would have lapsed before August 2 when Olivia purportedly revokes. Olivia's better argument is that she could revoke on August 2 because Grant's effort to accept via deposited mail was unreasonable under these circumstances. C is not a good answer. First, nothing in Olivia's offer suggests that she is making a promise to pay Grant in exchange only for his performance. Recall that an offer for a unilateral contract must make it very clear that the offeror will only take a counter performance and not a counter promise as acceptance. If the offer is unclear, it is presumed to invite acceptance by any reasonable means, which almost always authorizes an offeree to accept by a counter-promise. Second, Olivia specifically says that she needs a "response" from Grant. This strongly indicates that Olivia expects and wants Grant's promise to perform, making this an offer for a bilateral contract.

Henry sees his 10-year-old neighbor, Casey, playing outside on Friday afternoon. He walks over to her and says, "I'll pay you $30 if you actually mow my lawn tomorrow." Casey enthusiastically agrees to do so. She shows up on Saturday morning at 8 am and begins mowing the lawn. Casey is about halfway through mowing the lawn before Henry comes out and says, "Sorry, Casey, I've changed my mind. $30 is too much. I'll just mow it myself." Which of the following statements about this interaction is most likely accurate? A. Henry's statement is a breach. B. Henry is able to revoke his offer since Casey has not yet accepted. C. Henry is in breach, so Casey can stop mowing and collect the $30. D. Henry cannot revoke his offer because Casey has begun performance.

Henry cannot revoke his offer because Casey has begun performance. Again, Henry appears to have made an offer seeking performance as acceptance. Although usually offerors can revoke anytime prior to acceptance, to prevent offerors from taking advantage of offerees in unilateral contracts, an offeror loses the ability to revoke once an offeree has rendered substantial or part performance (depending on the jurisdiction). Restatement Section 45 tells us that "[w]here an offer invites an offeree to accept by rendering a performance and does not invite a promissory acceptance, an option contract is created when the offeree tenders or begins the invited performance or tenders a beginning of it." In other words, the offeror can't revoke once performance has begun. That makes D the best answer. A is not a terrible answer, but Henry is probably not yet in breach because Henry's payment (which is how he'll perform) isn't yet due. That Henry is not in breach also makes C not the best answer. If Henry has not yet breached, even if there is a good argument for anticipatory repudiation. Casey walking off the job means that Casey is actually in breach. Yikes! If Casey breaches she almost certainly cannot recover "on the contract" even if she might be able to pursue successfully some other theory of recovery.

Which of the following DOES NOT contain a 'condition' as that term is defined in the Restatement (Second) of Contracts? A. Athena tells Demeter, "If you will paint my house, I will pay you $1,000 on condition that 30 days have passed after you have finished." Demeter paints Athena's house. B. Artemis contracts to sell and Clea to buy goods pursuant to a writing which provides, under the heading "Conditions of Sale," that "the obligations of the parties are conditional on Clea obtaining from Xeres Bank by June 30 a letter of credit" on stated terms. C. Artie purchases land from Mrs. Hera, who is unable to get Mr. Hera to join her in signing the deed because they are engaged in divorce proceedings. Artie takes possession under a deed signed by Mrs. Hera, pays Mrs. Hera $10,000 and promises to pay an additional $5,000 "if, within one year, (1) Mr. and Mrs. Hera execute a quitclaim deed to Artie or (2) Mrs. Hera furnishes Artie with a certificate of the death of Mr. Hera with Mrs. Hera surviving him, or (3) Mrs. Hera as a single person executes a quitclaim deed to Artie after having been awarded the land following the entry of a final decree of divorce from Mr. Hera." D. Iris and Hecate contract to merge their corporate holdings into a single new company. It is agreed that the project is not to be operative unless the parties raise $600,000 additional capital.

The answer choice which does not contain a condition is answer choice A. The passage of time is not considered a condition, since a condition must be contingent and the passage of time is inevitable. (You can insert your own metaphysical observation here. For purposes of contractual conditions, however, my statement stands.) Each of the other answer choices contains a condition as explained in various Restatement Illustrations. Answer choice B is Illustration 1 from the Restatement (Second) of Contracts § 224. That section provides (with emphasis added) "1. A contracts to sell and B to buy goods pursuant to a writing which provides, under the heading "Conditions of Sale," that "the obligations of the parties are conditional on B obtaining from X Bank by June 30 a letter of credit" on stated terms. The quoted language is a term of the agreement, not a condition. The event referred to by the term, obtaining the letter of credit by June 30, is a condition." Answer choice C is Illustration 6 from the R2K Sec. 224. It provides (again, emphasis added) "A purchases land from Mrs. B, who is unable to get Mr. B to join her in signing the deed because they are engaged in divorce proceedings. A takes possession under a deed signed by Mrs. B, pays Mrs. B $10,000 and promises to pay an additional $5,000 'if, within one year, (1) Mr. and Mrs. B execute a quitclaim deed to A, or (2) Mrs. B furnishes A with a certificate of the death of Mr. B with Mrs. B surviving him, or (3) Mrs. B as a single person executes a quitclaim deed to A after having been awarded the land following the entry of a final decree of divorce from Mr. B.' The three enumerated events are alternative conditions and A's payment of $5,000 to Mrs. B becomes due if any of them occurs." Answer choice D is Illustration 7 and provides (emphasis added) "A and B contract to merge their corporate holdings into a single new company. It is agreed that the project is not to be operative unless the parties raise $600,000 additional capital. The raising of the additional capital is a condition of the duties of both A and B. If it is not raised, neither A's nor B's performance becomes due."

Ruby, who is 16 years old, enters into a contract to buy a custom skateboard. Ruby selects all the components of the skateboard, and customizes a design for the deck. Altogether, the parts and labor cost $450. Ruby and the skate shop owner sign a form that indicates the essential terms of their agreement. The skate shop owner tells Ruby the skateboard will be ready in a couple weeks. When it is time to pick up the skateboard, Ruby realizes she doesn't have enough money to pay for it. She calls the skate shop and tries to cancel her order. The shop tells her that they will try to sell the board to someone else, but that because the board was custom made for Ruby, the shop is unlikely to be able to get the full $450. The shop tells Ruby that Ruby will be responsible for the difference between the $450 contract price and the amount the shop is able to sell the board. Which of the following is most likely? A. Ruby will not be responsible for the difference in price between what the shop sells the board for and the contract price of $450 because Ruby can disaffirm the contract since she is a minor. B. Ruby will be responsible for the difference in price between what the shop sells the board for the contract price of $450m since that is the shop's expected damages. C. Ruby will be responsible for the difference in price between what the shop sells the board for and the contract price of $450, because the board was custom made for Ruby. D. Ruby will be responsible for the difference in price between what the shop sells the board for and the contract price of $450 because the shop owner had no reason to know Ruby was a minor.

The best answer is "A. Ruby will not be responsible for the difference in price between what the shop sells the board for and the contract price of $450 because Ruby can disaffirm the contract since she is a minor." This questions test the minor incapacity defense. The Restatement sets out in Section 14, "Unless a statute provides otherwise, a natural person has the capacity to incur only voidable contractual duties until the beginning of the day before the person's eighteenth birthday." A minor who disaffirms a contract is entitled to recover all of the consideration he has conferred and, in return, the minor is expected to restore as much of the consideration as remains in the minor's possession." In this fact pattern, Ruby has not yet taken possession of the skateboard, so she does not have to return consideration. The skate shop owner is not entitled to damages, so answer choice B is not the best answer. Certain contracts are not subject to a minor incapacity defense, but this contract does not fit within any of those exceptions (the exceptions include contracts that provide for the necessities of life that are not otherwise available to the minor through a parent or guardian; contracts in which a minor misrepresents his age; contracts in which the minor willfully harms the property that is the subject of the contract; and certain statutory exceptions. There is no exception for custom-made goods, so answer choice B is not the best answer. Answer choice D is not the best answer because although if Ruby had misrepresented her age, she would not be entitled to the minor incapacity defense, there is no evidence here that she misrepresented her age.

Sam Student needs to download new software so he can participate in his classes, which are suddenly taught in a distance-learning format due to a pandemic. Sam's school tells him the name of the software ("GoToClass"), and he opens the URL in his laptop's browser. As soon as he opens the browser, he sees a "CLICK HERE TO DOWNLOAD button." The software has a free version, but Sam wants the added features of the for-pay version, which is $9.99 a month for 12 months. Sam clicks to download, and after entering his credit card information and setting up a password, the software loads. About a week later, Sam receives an email confirming his purchase. The email welcomes Sam to the "GoToClass Family" and contains usage tricks and tips. The email has an attachment in PDF form titled "Attachment." There is no reference to the attachment in the body of the email. Sam did not click to open the attachment. Had Sam opened the attachment he would have seen language that by using GoToClass, users were agreeing to the company's limit on consequential damages. GoToClass is overwhelmed with the number of new users and its functionality is compromised. GoToClass promises users it is "aware of the problem and working toward a solution" but refuses to provide refunds and will not permit users to suspend their monthly payments or terminate their 12-month contracts. Sam struggles with GoToClass and he hates distance-based learning. He drops out of his program, and consequently has to back out of the job he had lined up. The job would have paid $80,000 per year. Since he drops out so late in the year, he cannot get a tuition refund. Tuition in his mortuary science program is $20,000 per year. Sam sues GoToClass for breach of contract. If Sam succeeds in his breach of contract claim, which of the following best describes the remedies to which Sam is likely entitled? A. Sam is likely to recover the monthly fee. Sam is unlikely to recover the tuition or lost salary amount since a court is unlikely to find those amounts foreseeable. B. Sam is likely to recover the monthly fee. Sam will not recover the tuition amount or the lost salary because limits on consequential damages are always enforceable. C. Sam is likely to recover the monthly fee. Since GoToMeeting causes Sam to lose out on the job Sam had lined up, Sam is also likely to recover the $80,000 salary. Sam is also likely to recover at least $20,000 of tuition. D. Sam is likely to recover the monthly fee. Since GoToMeeting causes Sam to lose out on the job Sam had lined up, Sam is also likely to recover the $80,000 salary. Sam is also likely to recover at least $10,000 (one semester) of tuition.

The best answer is "A. Sam is likely to recover the monthly fee. Sam is unlikely to recover the tuition or lost salary amount since a court is unlikely to find those amounts foreseeable." B is a strong answer except for the misstatement of law it contains -- limits on consequential damages are not "always" enforceable. C and D are not the best answers for similar reasons. Dropping out of his program and withdrawing from a job offer is unlikely to be foreseeable. As our textbook (and the Restatement explain) foreseeability is present if the loss arises either (a) in the ordinary course of events, or (b) as a result of special circumstances, beyond the ordinary course of events that the party in breach has reason to know. Sam will also likely run into a causation problem if he attempts to recover tuition and the salary. Sam dropped out of his program because he hated distance learning. It's not clear that he would have stayed in the program had the online platform been different (in other words, it's not clear that he would have stayed in the program had GoToMeeting not breached).

Barb owns a Bernese Mountain dog named Emmett. Bernese Mountain dogs are a large breed, and Barb's dog weighs about 100 pounds. One afternoon, Emmett escaped from his backyard enclosure. Meghan was driving down the highway and saw Emmett wandering near traffic. Meghan is a big fan of dogs, and stopped to see if she could help. Emmett happily got into Meghan's car, and Meghan took Emmett home with her. Unfortunately, the dog had been in a fight with a wild animal during his escapades, so Meghan took him to the veterinarian. Meghan incurred a $750 vet bill. Meghan tried to find Emmett's owner, but the dog did not have a microchip or other identifying information (no tags!). Meghan purchased dog food and a few other supplies at a cost of $100. About a month later, Meghan's efforts to find Emmett's owner finally paid off and she was able to reunite Emmett and Barb. Barb was so touched by Meghan's efforts and care for Emmett, that Barb promised to pay Meghan $500 to compensate Meghan for her time, effort, and expenses. What is the best characterization of Barb's promise to pay Meghan? A. Barb's promise will likely be enforceable under a theory of promissory restitution (or promise for a benefit received). B. Barb's promise will likely be enforceable because it is a contract implied in fact. C. Barb's promise will not likely be enforceable because Meghan is an officious intermeddler. D. Barb's promise will not likely be enforceable because past consideration and moral obligation cannot constitute consideration.

The best answer is "A. The promise is binding." Promissory restitution, or promise for a benefit received (PBR), requires the promisee to show a promise made in recognition of a benefit previous received. Here, we have a promise by Barb to pay Meghan for caring for Barb's dog. PBR is an express exception to the rules around past consideration and moral obligation, so D is not the best answer. C is not the best answer because there are no facts in the problem that suggest Meghan is an officious intermeddler. B is not the best answer because the interaction between Barb and Meghan is not a contract implied in fact. Recall a contract implied in fact is when the parties' conduct permits us to find a contract with consideration for the promise. One example of a contract implied in fact when you order a drink by nodding and signaling to the bartending. Another common example of a contract implied in fact is getting a haircut.

Gardener agreed to install a rain garden for a new homeowner, Bertha. Bertha included very specific details in their construction plans. The details were so distinct and difficult to implement that only an experienced gardener would have been able to successfully complete the project. Gardner told Bertha, "No worries, I've been doing this stuff for years. I will get the job done with no problem." Gardener sincerely believed he would be able to do it. Bertha paid Gardener the full $20,000 price in advance. After Gardener began the project, he realized he could not, in fact, build the building according to Bertha's specifications. Gardener met with Bertha and accurately told them he could not build the building and suggested that Bertha hire a more experienced gardener. Bertha replied, "I already paid you, and I want you to finish the job. You have taken up too much of my time, and it is too late to find somebody else." Given that Gardener cannot install the rain garden, is Gardener in breach of contract if he does not complete the job? A. Yes. Gardener is in breach of contract because impossibility that is personal to Gardner does not excuse non-performance of an agreed upon obligation. B. Yes. Gardener is in breach of contract because Gardner knew or should have known that he did not have the skills to install the rain garden. C. No. Gardener is not in breach of contract because he honestly believed he could complete the project and, as soon as he discovered he could not do so, he informed Bertha. D. No. Gardener is not in breach of contract because at the time the contract was made, he had no reason to believe that he could not do the job Bertha was requesting.

The best answer is "A. Yes. Gardener is in breach of contract because impossibility that is personal to Gardner does not excuse non-performance of an agreed upon obligation." Here is the explanation from the Learning Library: "Impossibility of performance of a contract is classified as an objective impossibility. Subjective impossibility, which is personal to the promisor and does not adhere in the nature of the act to be performed, does not excuse non-performance of a contractual obligation."

On Monday, Seller offered to sell his horse ranch to Buyer for $300,000. Seller told Buyer the offer would expire in five days. On Wednesday, Buyer's best friend and lawyer, Attorney, told Buyer that Seller had offered to sell the farm to Attorney. On Thursday, Buyer went to Seller and handed Seller a written document that said, "I accept your offer." Seller replied, "Sorry, you are too late. Late last night, I sold the ranch to Attorney." In an action by Buyer against Seller for breach of contract, will Buyer win? A. Yes, because Buyer has not proven that Attorney is a reliable source. B. Yes, because the information Buyer learned from Attorney was not inconsistent with Seller's offer to Buyer C. No, because the information Buyer learned from Attorney revoked Seller's offer to Buyer. D. No, because Seller sold the farm to Attorney before Buyer accepted Seller's offer.

The best answer is "B. Explanation: A revocable offer can be revoked directly any time before it is accepted. Seller failed to directly revoke his offer to sell the farm to Buyer, so direct revocation is not at issue here. An offer may also be revoked indirectly if the offeror takes an action inconsistent with going forward with the contract, and the offeree acquires reliable information about the offeror's revocation. R2K § 43. Here, the information Buyer received (that Attorney was offered the land), was not inconsistent with Seller's offer to sell the land to Buyer. A landowner may offer to sell land to multiple parties; that is different than actually selling land (a unique thing) to multiple parties. The result would have been different had Buyer learned on Wednesday that Seller had actually agreed to sell the [ranch] to Attorney (versus offering to sell the [ranch]). Answer "C: is not the best answer. Explanation: A revocable offer can be revoked directly any time before it is accepted. Seller failed to directly revoke his offer to sell the farm to Buyer, so direct revocation is not at issue here. An offer may also be revoked indirectly if the offeror takes an action inconsistent with going forward with the contract, and the offeree acquires reliable information about the offeror's revocation. Restatement (Second) of Contracts § 43. Here, the information Buyer received (that Attorney was offered the land), was not inconsistent with Seller's offer to sell the land to Buyer. A landowner may offer to sell land to multiple parties; that is different than actually selling land (a unique thing) to multiple parties. The result would have been different had Buyer learned on Wednesday that Seller had actually agreed to sell the farm to Attorney (versus offering to sell the farm). Answer D: is not the best answer. Explanation: A revocable offer can be revoked directly any time before it is accepted. Seller failed to directly revoke his offer to sell the farm to Buyer, so direct revocation is not at issue here. An offer may also be revoked indirectly if the offeror takes an action inconsistent with going forward with the contract, and the offeree acquires reliable information about the offeror's revocation. Restatement (Second) of Contracts § 43. Here, Seller selling the farm to Attorney would not have been sufficient to satisfy the test for indirect revocation; Buyer would also need reliable information about the sale before his acceptance of Cairo's offer. Answer A is not the best answer. This answer choice was not in the Westlaw quiz. This answer choice is not the best answer because even if Attorney was not a reliable source, the information received from Attorney is not inconsistent with Seller's offer to sell the land to Buyer.

In a contract for the construction of a 10-unit strip mall in the suburbs, Owner gave Builder a set of instructions that included detailed specifications. Builder constructed the building according to each specification, except for one. The specifications stated that the doors to the offices on the top floor use a certain brand of screw. However, Builder made an innocent mistake and used screws of a different brand than that specified. The screws used were identical in quality to the brand specified in the contract. Has Builder breached the contract? A. Builder has partially breached the contract but will not be liable for any damages because Builder substantially performed. B. Builder has partially breached and will be liable for any damages resulting from the breach. C. Builder has substantially performed so Builder has not breached the contract. D. Builder has not breached, but Builder will be liable for any diminution in value for using the different brand of screw.

The best answer is "B. Builder has partially breached and will be liable for any damages resulting from the breach." This question tests the concept of partial breach/substantial performance. Although use of the different brand of screw does not seem like a big deal, it is nonetheless a breach since the specifications indicated a particular type of screw. Since the builder has breached, answers C and D are not the best answers. A is also not the best answer, because substantial performance does not mean the breaching party (aka, the party that has substantially performed) is not responsible for damages.

organ is a tutor for law students and attorneys preparing to take the bar exam in the State of Hammitch. She signs a contract with Dudley to provide 20, individualized tutoring sessions in anticipation of the July bar exam in the State of Hammitch. The contract price is $5,000. The price is higher than other bar preparation services because Morgan creates custom practice questions for each student based on a diagnostic test and an individualized plan for that specific student. Dudley pays $500 down and takes Morgan's diagnostic test. Based on that test, Morgan creates hundreds of practice questions just for Dudley. Three weeks before the bar exam, and with 10 tutoring sessions remaining, the Board of Bar Examiners cancels the July exam and all future bar exams because the state suddenly moved to a diploma privilege. The board explains that diploma privilege means that any student who has graduated from an ABA approved school is admitted to the bar assuming the candidate satisfies the character and fitness requirements. Dudley is a graduate of an ABA approved school. If Morgan sues to enforce the contract, Dudley's best defense to Morgan's claim will be: A. Mutual Mistake B. Frustration of Purpose C. Misunderstanding D. Impossibility

The best answer is "B. Frustration of purpose." Frustration of purpose applies if, after contract formation, a supervening event occurs without fault of the party seeking relief and the event causes a party's performance to become substantially frustrated. The non-occurrence of the event was a basic assumption on which the contract was made. The party's duties are discharged unless the contract language or circumstances indicate risk should be allocated to one party. It can be hard to distinguish some of these defenses/excuses. Answer A. Mutual Mistake, is not the best answer because the parties did not make a mistake at the time of formation. When the contract was formed, the bar exam was required and there was no diploma privilege. Answer choice D is not the best answer because Morgan and Dudley could still go through with the tutoring -- it would just be pointless. Answer choice C is not the best answer because "misunderstanding" by itself is not one of the defenses/excuses to contract.

Which of the following is within the Statute of Frauds? A Myrtle orally promises to work for Grey, and Grey promises to employ Myrtle during Myrtle's life at a stated salary. B. Marie, a law professor, and Bertha, a first-year, first-semester law student, orally agree that Marie will employ Bertha as her research assistant until Bertha graduates from law school. The student handbook contains the following provision: "In no circumstance will a student be permitted to complete the course of study for the J.D. degree earlier than 24 months or later than 84 months after the student has commenced law study at the law school." C. Henry and Ed orally agree that Henry will grade all of Ed's Contracts essay exams as long as Ed is a law professor. Ed is a tenured member of the law faculty, and Ed has no immediate plans to retire. Each year, the associate dean assigns law professors to teach classes based in part on the interest of the professor and in part on the needs of the school. D. None of the above are within the Statutes of Frauds.

The best answer is "B. Marie, a law professor, and Bertha, a first-year, first-semester law student, orally agree that Marie will employ Bertha as her research assistant until Bertha graduates from law school. The student handbook contains the following provision: "In no circumstance will a student be permitted to complete the course of study for the J.D. degree earlier than 24 months or later than 84 months after the student has commenced law study at the law school." A contract is within the statute of frauds if, by its terms, the contract is impossible to be performed within one year. Lifetime contracts are not within the statute of frauds because none of us is guaranteed to live beyond a year (though we all hope for long and healthy lives), therefore answer choice A is not the best answer. Answer choice C is not the best answer because although Ed does not have any immediate plans to retire, there is nothing in the contract making it impossible for the contract to be performed within one year. This example is loosely analogous to the railroad grading and ties illustration in R2K 130. In contrast, answer choice B cannot, by its terms, be performed within one year because Bertha cannot graduate from law school within one year. This example is similar to the Restatement Section 130, Illustration 4 (the one about five crops of potatoes).

Negotiations between Konar-Steenberg and Holcomb concerning the sale of Holcomb's lake property culminated on May 1 when Holcomb signed the following document: "In consideration of the sum of $10 paid and received, I offer to sell my property on Clear Lake to MKS for $50,000 cash. This offer to remain open until 5 p.m. on May 15." /Signed Holcomb Holcomb attempted to revoke her offer in a letter mailed on May 3, which never was received by Konar Steenberg. On May 12, Konar Steenberg mailed an acceptance that was received by Holcomb on May 16. In an action by MKS to enforce the contract, a court most likely will rule that: A. No contract was formed because the May 3 revocation was effective on dispatch despite never being received by Konar Steenberg. B. No contract was formed because option contracts are not binding unless supported by consideration. C. No contract was formed because the acceptance was not received until May 16, one day beyond the date for acceptance. D. A contract was formed because the revocation was ineffective and the acceptance was effective on disptach.

The best answer is "C. No contract was formed because the acceptance was not received until May 16, one day beyond the date for acceptance." This is an option contract; recall that the rules for acceptance of option contracts differ. R2K§ 63. Time When Acceptance Takes Effect, "Unless the offer provides otherwise, (a) an acceptance made in a manner and by a medium invited by an offer is operative and completes the manifestation of mutual assent as soon as put out of the offeree's possession, without regard to whether it ever reaches the offereor; but (b) an acceptance under an option contract is not operative until received by the offeror. (emphasis added). Even if you did not recall the option contract rule, recall that the mailbox rule is a default rule - typically, an acceptance is effective upon dispatch (here that would be when MKS mails the letter). The offeror has the power, however, to dictate that an offer be received by a particular date to be effective. Here, Holcomb specified the offer expired at 5 p.m. on May 15. MKS had until 5 p.m. to communicate his acceptance to Holcomb. A is not the best answer because it mistakes the default rule for revocation (revocation is typically effective upon receipt, not upon dispatch). B is not the best answer because the language of the problem indicates there was consideration for the option. D is not the best answer because even if the revocation was "ineffective" the acceptance was not received before the offer lapsed by its terms.

Sam Student needs to download new software so he can participate in his classes, which are suddenly taught in a distance-learning format due to a pandemic. Sam's school tells him the name of the software ("GoToClass"), and he opens the URL in his laptop's browser. As soon as he opens the browser, he sees a "CLICK HERE TO DOWNLOAD" button. The software has a free version, but Sam wants the added features of the for-pay version, which is only $9.99 a month for 12 months. Sam clicks to download, and after entering his credit card information and setting up a password, the software loads. About a week later, Sam receives an email confirming his purchase. The email welcomes Sam to the "GoToClass Family" and contains usage tricks and tips. The email has an attachment in PDF form titled "Attachment." There is no reference to the attachment in the body of the email. Sam did not click to open the attachment. Had Sam opened the attachment he would have seen language that by using GoToClass, users were agreeing to arbitrate any dispute with the company and were agreeing to the company's limit on consequential damages. GoToClass is overwhelmed with the number of new users and its functionality is compromised. GoToClass promises users it is "aware of the problem and working toward a solution" but refuses to provide refunds and will not permit users to suspend their monthly payments or terminate their 12-month contracts. Sam, along with many other students, joins a class action to get out of the contract. GoToMeeting moves to compel arbitration. If the software company moves to compel arbitration, what is Sam's best argument? A. The motion to compel should be denied because Sam's school forced him to use GoToClass. B. The motion to compel should be denied because Sam was subject to undue influence. C. The motion to compel should be denied, because Sam did not have notice of the terms, and therefore did not manifest his assent to the term. D. The motion to compel should be denied because arbitration effectively denies Sam a forum.

The best answer is "C. The motion to compel should be denied, because Sam did not have notice of the terms, and therefore did not manifest his assent to the term." This fact pattern is similar to that in Specht v. Netscape Communications Corp. (p. 240) in which the Second Circuit held that parties were not bound by an arbitration agreement that the parties were not aware of prior to executing the download. As Justice (then Judge) Sotomayor wrote in the opinion, "[m]utual manifestation of assent, whether by written or spoken word of by conduct, is the touchstone of contract." In this question, just as in the Specht case, there was no "immediately visible notice of the existence of license terms" and there was no requirement that Sam provide "unambiguous manifestation of assent." Correct AnswerCorrect Answer B is not the best answer: "The motion to compel should be denied because Sam was subject to undue influence." Undue influence is defined as "unfair persuasion of a party who is under the domination of the person exercising the persuasion or by virtue of the relation between them is justified in assuming that that person will not act in a manner inconsistent with his welfare." R2K Section 177. Undue influence almost always involves a fiduciary or confidential relationship during which the fiduciary (or dominant person) substitutes their own will for that of the influenced person's will. This fact pattern does not support an undue influence defense. Note we did not spend much class time on this defense, and it's unlikely that a topic we did not spend class time on would be the best answer. A is not the best answer. The fact that GoToClass was required by the school is not Sam's best argument for refusing to enforce the arbitration agreement. Sam's agreement with GoToClass is independent of his arrangement with his school.

Horse Trainer (Trainer) wanted a piece of expensive equipment for a new training method he wanted to try. Friend, who is Trainer's oldest and closest friend, happened to possess a vast stock of horse training equipment at his home so Trainer paid him a visit. Friend was excited to see the trainer and offered to give Trainer all the equipment he wanted free of charge. Trainer replied, "While I am grateful for your generosity, I still think we should write up a contract that way you don't take me to court later complaining that we don't have a binding contract." Friend understood and the two parties drafted and signed a simple, written agreement that provided, "Friend will deliver to Trainer the horse training equipment specified below in exchange for good and valuable consideration, including the many occasions on which Trainer has provided free trail rides to Friend and Friend's family, and $10." Has Trainer provided consideration for Friend's promise? A. Yes. A court will not inquire into the adequacy of consideration where, as here, the parties have expressly agreed in writing that their exchange involves adequate consideration. B. Yes. The many occasions on which Trainer has provided free tickets to his shows to Friend suffices as consideration. C. Yes. Trainer's promise to pay $10 demonstrates how much these parties have valued the equipment and therefore serves as consideration. D. No. Horse Trainer did not provide valid consideration.

The best answer is "D. No. Horse Trainer did not provide valid consideration." As the Westlaw Learning Library explains, "[a] mere recital that the parties' promises are good consideration does not bind a court to conclusion that the parties exchanged consideration. [Trainer's] past generosity ... is past consideration, which is not valid consideration. $1[0] is sham or nominal consideration and therefore is not good consideration, especially in light of the disparity of values exchanged. See Rest. 2d § 79, cmt. d." "A Yes. A court will not inquire into the adequacy of consideration where, as here, the parties have expressly agreed in writing that their exchange involves adequate consideration" is not the best answer. Again, the Learning Library explanation, "A mere recital that the parties' promises are good consideration does not bind a court to conclusion that the parties exchanged consideration." "B. Yes. The many occasions on which Trainer has provided free tickets to his shows to Friend suffices as consideration" is also not the best answer." As Learning Library explanation provides, "[a]s the [Trainer's] past generosity in allowing Friend to attend [Trainer's] shows for free is past consideration, which is not good consideration." Finally, "C. Yes. Trainer's promise to pay $10 demonstrates how much these parties have valued the equipment and therefore serves as consideration" is also not the best answer because "$[10] is sham or nominal consideration and therefore is not good consideration, especially in light of the disparity of values exchanged."

Rebecca is a world class skier who enters into a contract with Folsom Custom Skis for a set of custom downhill skis for $5,000. Rebecca orders the skis in July, and expects them to be ready by the time the next ski season begins in early November. A few months after she signs the contract, Rebecca decides she can no longer afford the skis and repudiates the contract. Assume this is a total breach of contract. Folsom has already purchased the materials to build the custom skis for Rebecca; Folsom spent $2,000 on materials that have been used in Rebecca's skis. Folsom has paid its technicians $1,000 to begin work on the skis; to finish, Folsom will have to pay an additional $500 in labor costs. Which of the following best describes Folsom's likely damages in its breach of contract action against Rebecca? A. Since Folsom is the nonbreaching party, Folsom can recover the contract price, which is $5,000. B. Folsom's recovery is limited to the amounts Folsom has already spent, since Folsom is required to mitigate. Therefore Folsom will be awarded $3,000 (this is the $2,000 in materials and the $1,000 in labor costs but not the $500 in anticipated labor costs). C. Folsom is entitled to the benefit of its bargain. Had the contract been performed, Folsom would have made a profit of $1,500 (this amount represents the contract price of $5,000 less the $3,500 Folsom would have spent to build the skis). Folsom therefore can recover $1,500. D. Folsom is entitled to the benefit of its bargain, and Folsom is also required to mitigate. Had the contract been performed, Folsom would have made a profit of $1,500 (this amount represents the contract price of $5,000 less the $3,500 Folsom would have paid to build the skis). Folsom will be entitled to recover $1,500 less whatever loss Fulsom can avoid through mitigation. The problem does not provide sufficient information about Folsom's efforts to mitigate.

The best answer is "D." A is not the best answer. The goal with expectation damages is to put the nonbreaching party (here Folsom) in the position it would have been in had the contract been performed. The contract price is not the correct measure of where Folsom would have been had the contract been performed. Awarding the contract price here overcompensates Folsom, since Folsom retains the skis. A does not address Folsom's duty to mitigate. B is not the best answer because it contains an incorrect statement of law. It is true that Folsom is required to mitigate, but that requirement does not mean that Folsom is limited to the amounts it has already spent. C is getting closer, but C leaves out the requirement to mitigate. That leaves us with D as the best answer.

Mara owns a coffee shop and has a dozen employees, most of whom are part-time. Mara decides to sell the coffee shop so she can travel the world. Mara is concerned about finding a buyer because the pandemic has been hard on business. Mara knows it will be easier to find a buyer if the coffee shop is fully staffed. Prior to finding a buyer, Mara tells her employees, "I promise to give each of you a bonus of two-weeks wages if you stay on until someone purchases the company." To Mara's surprise and delight, within a week, Mara finds a buyer. Is there consideration for Mara's promise to her employees? A. Yes. There is consideration for Mara's promise to her employees. B. No. There is not consideration for Mara's promise. C. Although there is consideration for Mara's promise, there is no consideration for the employees. Since there must be consideration on both sides, there is a failure of consideration. D. Since she found a buyer so quickly, Mara does not have to giver her employees the bonus since they did not suffer a detriment.

The best answer is A. There is consideration for Mara's promise. Mara will benefit from being full staffed (the problem says she knows it will be easier to find a buyer if she's fully staffed). In addition, because the employees are not under an employment contract they have no legal obligation to stay on at the coffee shop. Staying on, then, is a waiver of a legal right; therefore an employee who stays has incurred a detriment. Since we can find a benefit for Mara and a legal detriment for the employees, we have the requirements of consideration. While it is true that an enforceable contract requires consideration on both sides, since there is consideration for Mara and for the employees, neither B nor C is the best answer. D is not the best answer because even though the detriment (not leaving the job) does not last long, it is adequate consideration. Remember that so long as there is sufficient consideration, a court will not inquire into the adequacy of the consideration

Micah has a contract to work for Employer for 12 months. The contract provides that Micah will be paid each month, and her salary is to be $120,000, which is $10,000 per month. After 4 months on the job, Micah's boss fires her. Although a breach of contract, assume the termination does not violate any state or federal employment law. When Micah was fired, she had been paid $40,000 for the 4 months she worked. Micah is devastated. She immediately begins looking for another job, but cannot find anything but entry-level service jobs in the first three months. In month four, Micah hires a placement consultant (a career counselor) for $1,000. Finally, Micah finds a job after looking for another three months. At her new job, she earns $11,000 per month. Micah sues her former employer for breach of contract. Which of the following best explains Micah's likely damages award? A. Micah is likely to recover $58,000, which represents the difference between what she would have received had the contract been performed, $120,000, and the sum of the payments received by her new employer and her former employer--$40,000 from former employer and $22,000 from new employer. B. Micah is likely to recover $59,000, which represents the difference between what she would have received had the contract been performed, $120,000, and the sum of the payments received by her new employer and her former employer, which was $40,000 from former employer and $22,000 from new employer, plus the $1,000 she paid for career counseling. C. Micah is likely to recover the remaining salary due under the contract, which is $80,000 ($120,000 contract price less $40,000 payments received). D. Micah is unlikely to recover any amount of damages since she was obligated to mitigate by securing alternate employment.

The best answer is B because B describes the expectation damages Micah is likely to recover in a successful breach of contract action against her employer. A is close, but omits the expenses Micah incurred in her efforts to mitigate. Micah's decision to hire a career counselor is a reasonable effort to mitigate. Answer choice C is not the best answer, because it does not factor in Micah's mitigation by finding alternate employment. Awarding Micah $80,000 would overcompensate Micah. Answer choice D is also not the best answer because it would undercompensate Micah. Although Micah found another job, she lost out on several months of salary. Micah is entitled to that lost salary even though she successfully mitigated some of her damages.

Professor Epstein, a professor at another law school, and Professor Shea oral agreed on the following terms: "Professor Shea agrees to completely grade Professor Epstein's Contracts Essay exams by 5 p.m. on June 5. Professor Epstein will pay $150 per exam." Before the parties wrote up their agreement, the Vice Dean at Professor Epstein's school changed the due date for exams, and Professor Epstein's exams were now due by June 1. Consequently, Epstein needed his exams graded by June 1. Epstein and Shea then signed a written contract; it provided, "Shea agrees to grade Epstein's exams by 5 p.m. on June 1. Epstein will pay $150 per exam." The writing included no other terms—just the parties' signatures. On May 29, Epstein asked Shea how she was doing on the grading. Shea responded that she would have the exams done by June 5. Epstein told Shea that he needed the exams by June 1, per the parties' contract. Citing the oral agreement, Shea insisted that she had until June 5 to grade the exams. May Shea introduce the oral agreement to show the exams had to be graded by June 5? A. No. Shea cannot introduce the earlier agreement because the later agreement was completely integrated. B. No. Shea cannot introduce the earlier agreement because it directly contradicts the later agreement. C. Yes. Shea may introduce the earlier agreement because the prior agreement is consistent with the subsequent written agreement. D. Yes. Shea may introduce the earlier agreement to help show the meaning of terms in the later agreement.

The best answer is B. Here is the answer explanation from the quiz, modified to fit our facts: "Where there is a binding agreement, either completely or partially integrated, evidence of prior or contemporaneous agreements or negotiations is not admissible in evidence to contradict a term of the writing. See R2K § 215. A binding integrated agreement discharges inconsistent prior agreements, and evidence of a prior agreement is therefore irrelevant to the rights of the parties when offered to contradict a term of the writing. See R2K §215, cmt. a, 'An earlier agreement may help the interpretation of a later one, but it may not contradict a binding later integrated agreement.' See R2K § 215, cmt. b. The initial agreement between the parties contradicted the term dealing with date in the later integrated agreement (June 1 versus June 5). Therefore, the evidence cannot be admitted."

Arthur leases his recreational vehicle to Barbara for the summer. Barbara is planning to do the Great Lakes "circle tour" during the months of June and July. Barbara has a signed option from Arthur to purchase the R.V. for $50,000 in cash any time before August 1 (the day the RV is due back to Arthur). The option was supported by consideration. On July 30, Barbara phones Arthur and asks if he would accept $39,000 in cash for the RV. Neither party is a merchant. Which of the following is the most accurate explanation of the result of Barbara's July 15 phone call? A. Barbara's phone call constitutes a counter-offer, and Authur's original offer is therefore terminated. B. If Arthur refuses to accept the #39,000 Barbara can still accept the original offer/deal. C. Barbara's phone call constitutes a rejection and she can no longer accept unless Arthur renews his offer. D. Barbara's phone call constitutes a counter-offer, and since the option is so close to expiration, Arthur will be permitted to sell the RV to another without breaching the agreement.

The best answer is B. Option contracts, unlike regular offers, do not terminate by the usual offer termination events (rejection, counteroffer, revocation, death/incapacity of offeror). This problem is based on Illustration 1 to Restatement § 37. Termination of Power Of Acceptance Under Option Contract. The provision provides that "the power of acceptance under an option contract is not terminated by rejection or counter-offer, by revocation, or by death or incapacity of the offeror, unless the requirements are met for the discharge of a contractual duty." Illustration 1: A leases land to B, giving B an option to purchase the land for $10,000 in cash during the term of the lease. Misinterpreting the lease, B attempts to exercise the option by tendering a mortgage for $10,000. A refuses to accept the mortgage. B retains power to exercise the option by a tender conforming to the terms of the lease.

Holcomb decides she has had it with law teaching and is going to pursue her dream of teaching preschool. She calls New Horizons, a childcare center franchise, to inquire about purchasing a franchise. She talks to the Vice-President, who tells her, "We only grant franchises to individuals who can show a net worth of $500,000, and who have a degree in early childhood education. If you just had the money and the degree, you would be a great franchisee candidate. You know, you can get the degree online through the University of West Tennessee." Holcomb has neither the money nor the requisite degree. She decides this will have to be a three-year process; she gives up her tenure and works for three years as a tax associate at a big downtown firm. She socks all her salary and bonus away so that that she can show the requisite net worth. Simultaneously, she enrolls in the University of West Tennessee's online associate's degree program in early childhood psychology. She earns her associate's degree. In year four Holcomb goes back to New Horizons, but they refuse to grant her a franchise. She cannot get her law teaching job back, and she does not want to return to the law firm. If Holcomb sues New Horizons on grounds of promissory estoppel, New Horizon's best argument would be: A. Holcomb did not get the degree specified: she got a degree in childhood psychology, but they require early childhood education. B. There was no promise. C. There was no reliance. D. Holcomb did not confer a benefit on New Horizons, so it is not unjust for them not to award her a franchise.

The best answer is B. There was no promise. Recall that to succeed in a promissory estoppel claim, the promise must be sufficiently clear and definite. Here, New Horizon's language is not clear and definite; New Horizons tells Holcomb "you would be a great candidate" which is not a promise to grant a franchise. Answer choice C. (There was no reliance.) is a tempting answer, but not as good as B. Holcomb will argue that she in fact did rely because she quit her job, incurred the expense of getting a degree, etc. It's possible that Holcomb's reliance was not reasonable, but that is not the language of the answer choice. It is also not as strong of an argument as the straightforward - -there is no promise argument. Answer choice D is not the best answer because it conflates restitution theory with promissory estoppel. Recall that in promissory estoppel our focus is not on the benefit the promissor received, but on the detriment the promisee suffers. The opposite is true in unjust enrichment, which we will study more at the end of the term - focus on the value of the benefit received.

Several law students were drinking at a bar (assume we are in a post-COVID world) and discussing their favorite torts professor. The students talked about playing a (harmless and not too mean) practical joke on the professor. Student A, laughing, said, "I would give you $1,000 if you actually did that in class. That would be hilarious." The next day in class, Student B actually pulled off the practical joke. If Student B seeks to enforce the promise to pay $1,000, what is the most likely result? A. Student A does not owe Student B the money if Student A was joking. B. Student A owes Student B the money if (1) a reasonable person in Student B's position would have believed Student A was serious and (2) Student A was in fact serious. C. Student A does not owe Student B the money because practical jokes in law school are against public policy. D. Student A does not owe Student B the money because the students were drinking and were not competent to form contracts. E. Student A owes Student B the money if a reasonable person in Student B's position would have believed A was serious.

The best answer is B. This challenging question tests the concept of mutual assent. A is incorrect because it is not enough that one party to the contract formation was joking (remember Lucy v. Zehmer?). C is incorrect because although practical jokes against professors are a bad idea, contracts premised on practical jokes are not unenforceable as contrary to public policy. D is incorrect because although a high degree of intoxication could prevent contract formation due to incapacity or incompetence, here there are insufficient facts to establish the students were not competent. E. Is really close, but still wrong because both parties know they are joking, so it doesn't matter if a reasonable person would have thought A was serious. B is a better answer.

Jay and Daisy negotiated a contract under which Jay would purchase Daisy's vintage typewriter for $5,000; the typewriter was rumored to be the typewriter on which a famous author wrote their best known novel. After discussing the terms for several weeks, the parties signed a multiple page, written agreement for sale that detailed price, payment terms, delivery terms, inspections rights, warranties, and numerous other matters. The contract included a merger clause. The next day, Jay telephoned Daisy and asked if Daisy would, as part of the deal, give Jay writing lessons. Daisy agreed to give Jay two-hour writing lessons on each of the next five Sunday mornings. Daisy later wants to renege on her promise to give Jay writing lessons. What is the most likely outcome if Jay seeks to enforce the promise to give writing lessons? A. Daisy will likely prevail because the writing lesson agreement was not in writing. B. Daisy will likely prevail as long as she raises a parol evidence rule objection to the admission of the oral agreement. C. Jay will likely prevail even if Daisy raises a parol evidence rule objection to the admission of the oral agreement. D. Jay will likely prevail because the agreement was only partially integrated.

The best answer is C. As the MCQ explanation provides (modified to fit our facts): "The parol evidence rule is inapplicable to a subsequent agreement, such as Daisy's promise to give the writing lessons. Under either the UCC or the common law, application of the parol evidence rule ends when the parties sign the contract; enforceability of any post-signing agreement (written, oral, with or without consideration) is judged independently of the parol evidence rule." A is not the best answer because there is no requirement that the writing lesson agreement be in writing. It does not fall within any of the Statute of Frauds categories. B is not the best answer because the parol evidence rule is inapplicable to a subsequent agreement, such as Daisy's promise to give the writing lessons. D is not the best answer because regardless of the level of integration, the parol evidence rule does not apply to subsequent agreements.

A fancy (and expensive) fitness center opens near Mitchell Hamline's campus. One of your classmates requests an application to join the fitness center, which is open only to members. The center sends your classmate the application, which the classmate completes and returns to the fitness center. The application required your classmate to fill in personal information and an emergency contact. The application required your classmate to sign stating that all of the information was truthful. The fitness center confirmed receipt of the application. Which of the following best describes the parties' relationship at this point? A. When your classmate returned the application, the classmate promised to enroll in the fitness center. B. The fitness center has made a promise to enroll your classmate. C. Your classmate has not made a promise to enroll and the fitness center has not made a promise to accept your classmate. D. Both your classmate and the fitness center have made promises: your classmate promised to enroll if accepted and the fitness center promise to accept your classmate.

The best answer is C. Neither the fitness center nor your classmate made a promise. Applications suggests that a request is being made from someone in authority who had the power to grant or deny the application. Accepting an application is not a promise that the applicant will be accepted. Likewise, the only promise that your classmate made by signing and sending in the application was that the information contained inside by true. In this question, there is no suggestion that the "application" was a promise to enroll if accepted. The Restatement (Second) of Contracts §2(1) defines a promise as "a manifestation of intention to act or refrain from acting in a specified way, so made as to justify a promisee in understanding that a commitment has been made."

Thelma and Louise live together before Louise starts law school. Thelma quits school (she has been working on a graduate degree in mechanical engineering) and works full time as a bartender to put Louise through law school. At Louise's law school graduation party Louise stands up and acknowledges how she could never have gotten through law school without Thelma's emotional and financial support. She continues: "Now that I have gotten my J.D. and landed a great job at Big Law, I'll be able in some small measure to pay her back by letting her finish her degree. I promise that I'll put her through school just like she put me through school." Thelma, rendered nearly speechless by Louise's moving tribute, says, "Thank you! I'll go back to school next fall." Two weeks later, however, Louise meets Doris, a paralegal at the new firm, falls in love, and moves out. She now refuses to pay for Thelma's education. If Thelma sues Louise for breach of contract, Louise's best argument is likely to be: A. Thelma cannot earn her degree in less than two years, and the contract is not in writing, so it is not enforceable. B. No reasonable person would think Louise was making a serious promise. C. The promise was gratuitous, in recognition of benefits already received, and thus there is no consideration for it. D. Thelma has not relied to her detriment on the promise.

The best answer is C. The bargain theory of contract requires consideration and if the promisor made the promise because of a past performance, there can be no bargain. My contracts professor used the phrase, "past consideration is no consideration." A promise based on consideration received in the past is generally unenforceable since it was not bargained for. Our textbook walks us through the rationale for why this is so at page 39. Thelma might have a colorable argument for some relief, but she does not have a good argument under a contract theory that relies on consideration.

Leigh was downsizing and wanted to sell most of her belongings. Rather than sell things item-by-item, Leigh wanted to hire someone to do an estate sale and sell all her things in one fell swoop. Caleb had an estate sale company and was considering paying Leigh a lump sum for all of the items. Caleb was not sure if he wanted to purchase all of Leigh's belongings. The holidays were fast approaching, and Caleb asked Leigh if she would give Caleb time to think about it. Leigh agreed, and Leigh signed the following, "In consideration of the sum of $10 paid and received, I offer to sell to Caleb the items identified as 'Estate Sale' in my home for an amount not less than $2,000 or a higher amount if an independent appraiser (selected by both parties) warrants that the items are worth more than $2,000. This offer to remain open until January 10." Signed Leigh Shortly after signing this agreement, Leigh decided that she was better off selling the items all herself. She called Caleb right after Christmas (well before January 10) and left a voicemail for Caleb informing him of her decision. On January 9, Caleb called Leigh and told her that he was accepting the deal. In an action to enforce the contract, a court most likely will rule that: A. There is no contract because Leigh's voicemail right after Christmas was an effective revocation. B. There is no contract because an option contract cannot be created for the sale of goods. C. There is a contract because Caleb's January 9 acceptance was with the time specified in the option. D. There is not a contract because we do not know if the offer is on reasonable terms.

The best answer is C. There is a contract because Caleb's January 9 acceptance was within the time specified in the option. This question tests your understanding of option contracts. R2K § 87 provides that "[a]n offer is binding as an option contract if it (1) is in writing and signed by the offeror, recites a purported consideration for the making of the offer, and proposes an exchange on fair terms within a reasonable time; or (2) is made irrevocable by statute." In this fact pattern, there is an offer in writing, signed by the offeror (Leigh) which recites a purported consideration ($10). We do not know if $2,000 is "reasonable terms" but the alternative option of an independent appraiser satisfies any concern about reasonableness. (So D is not the best answer.) A is not the best answer because option contracts are "indestructible" (as one of our colleagues put it). Unlike "regular" offers, option contracts survive attempted revocations, so Leah's purported revocation via voicemail was not effective. B is not the correct answer because there is no such prohibition. You might be thinking of Firm Offers, pp. 105-06 of our textbook. The "firm offer" rule applies to merchants. This hypothetical is better analyzed as an option contract, rather than a firm offer. The offer is from Leigh, and we have no indication Leigh is a merchant.

Aria told Fido, "If you promise to give me $1,000 to initiate a lawsuit against this Eval Corp., I will give you $50,000 when I recover from Eval." Fido accepted this offer from Aria but is afraid that it is not a valid contract because of the unequal values of the consideration. Is Aria's promise to pay $50,000 out of her lawsuit proceeds valid as a consideration for a promise of $1,000? A. No. Since Aria might not recover anything from Eval (Aria might lose the lawsuit), she's not supplying any consideration in exchange for Fido's promise. B. No. Aria has, in effect, promised to give Fido a gift of $49,000 and gifts are not consideration. C. Yes. Fido's promise is valid as consideration for Aria's promise. D. Maybe. Fido's promise may be valid as consideration for Aria's promise so long as a court concludes that, despite the differential in values, the exchange is fair.

The best answer is C. This question is similar to the deal the parties struck in Embola v. Tuppela (p. 305). This exchange passes the bargained-for-exchange test. Aria is seeking the $1,000 in exchange for her promise of $50k in the future, if she wins. Fido is giving the $1,000 promise in order to get the $50k promise. Fido's promise induced, at least in part, Aria's promise and Aria's promise induced, at least in part, Fido's promise. Because the promises are reciprocal inducements for one another, we have a bargained for exchange. A is not the best answer. The fact that the $50k may never get paid to Fido is, in fact, why the imbalance of the exchange may be more reasonable than it seems. Fido is taking a gamble on the chance at $50k, not actually $50k. B is not the best answer. Aria isn't offering a gift to Fido. She is seeking an exchange. It is true that sometimes radical imbalances in the consideration--what's sometimes referred to as "sham consideration"--can raise the possibility that the key promisor is really just disguising a gift in the clothing of a contract. But $1,000 isn't an insignificant sum (it's not "nominal"), there's no indication of a relationship between Fido and Aria that would suggest that Aria has some reason to give Fido a gift of that magnitude, Aria has specifically conditioned her promise on the receipt of the $1,000 promise from Fido, and this actually looks like a commercial gamble. Aria may need financing to help her initiate her lawsuit now. She may well have little or nothing else to offer Aria, and she might well be willing to offer a stake in the ultimate winnings of the case to induce Aria to invest $1,000 now. This basic bargain, in fact, is precisely what attorneys who work on a contingency fee basis do with clients all the time. In some cases, the bargain pays off and pays off big. People are free to take such gambles and the law will enforce their bargains to do so, even if the deal looks bad or risky or like a bargain the judge wouldn't strike were she in the parties' shoes. And that, of course, highlights why D is wrong. Courts are not in the business of judging, as a general matter, the fairness of the bargains that people strike.

Two acquaintances, Link and Zelda, were drinking at a bar in Uptown. Link had always admired Zelda's car, and in fact Link had offered to purchase Zelda's car in the past, but Zelda had always declined to sell it. After several drinks, and a lengthy discussion of how great Zelda's car was, Link said, "You know I'll give you $50,000 for your car." Zelda scoffed, "You don't have $50,000!" Link protested that she could put the money together within two days, and wrote out the following on the bar napkin, "I'll pay you $50k for your car. /s/ Link." Zelda shrugged, and took the napkin and stuck it in her pocket. Link's face broke into a big grin and Link said, "It's a deal, then!" If Link asks a court to force Zelda to sell the car to Link in exchange for $50,000, what is Link's best argument that there is a contract? A. Under the objective theory of contract formation, there was a manifestation of mutual assent when Zelda shrugged and took the napkin. B. Under the subjective theory of contract formation, Zelda probably intended to enter into the contract when Zelda shrugged. C. Zelda was not sufficiently drunk to avoid the contract. D. Link's offer was written out and signed.

There might not be a contract here, but Link's best argument is "A." Under the objective theory of contract formation, there was a manifestation of mutual assent when Zelda took the napkin. B is not the best answer because we do not follow a subjective theory of contract formation. C is not the best answer because although intoxication can be a defense, the fact that someone is not drunk is not, by itself, sufficient to enforce a contract. D is not the best answer because although it is helpful that the offer was written, the issue here is not whether Link made an offer but whether Zelda accepted. In addition, the question asks for the best argument for Link. Link needs to demonstrate that Zelda accepted, so Link's writing is not the relevant inquiry.

Owner, who owns the HarGar shopping mall, entered into a lease with Amazing Lash Studio (ALS). ALS is a beauty salon specializing in eyelash extensions. (Eyelash extensions are semi-permanent fibers that are attached to your natural eyelashes in order to make your lashes look longer, fuller, and darker.) The contract included the following two provisions, among others. 1) Amazing Lash Studio shall have the exclusive right to operate a beauty salon. 2) The rent shall be $1,500 per month. The contract did not contain any other relevant provisions, and the parties did not discuss either provision. During the term of the lease, Owner acquired land across the street from the mall and built a second shopping mall linked to the first mall by a bridge. Owner leased part of the new mall to BrowChic Nails and Lashes, a salon offering eyelash extensions as well as other services such as nail and hair care. ALS sued Owner for breach of contract. What is the most likely outcome of ALS's lawsuit? A. ALS likely will not prevail because Owner did not breach the exclusivity clause of the contract since BrowChic does not provide exclusively eyelash services. B. ALS likely will not prevail because Owner did not breach the duty of good faith and fair dealing. C. ALS will likely prevail because Owner breached the provision of the lease agreement that provided that "Amazing Lash Studio shall have the exclusive right to operate a beauty salon." D. ALS will likely prevail because Owner breached the duty of good faith and fair dealing.

The best answer is D. As the explanation in our Westlaw Learning Library provides, "Every contract imposes upon each party a duty of good faith and fair dealing in its performance and its enforcement. Restatement (Second) of Contracts §205. Illustration 2 is almost identical to the facts stated in this problem, and the drafters concluded 'Unless such action was contemplated or is otherwise justified, there is a breach of contract by A.' Restatement (Second) of Contracts §205, Illustration 2. Here, by building a mall across the street linked by bridges between its first mall and leasing to another [beauty salon] , Owner unmistakably undermined [ALS's] sales in a way that makes [ALS's] exclusive right almost meaningless." C is a tempting answer, but not the best answer because, as explained by the Learning Library, "the agreement is silent about building a new mall across the street and leasing that space to a competitor."

Holcomb and Butterfoss agree that Holcomb will purchase Butterfoss's foosball table for $450. They memorialize their agreement with the following language, signed by both: Holcomb agrees to buy, and Butterfoss agrees to sell, Butterfoss's foosball table. Cash payment of $450 upon pick-up. s/Holcomb s/Butterfoss Which of the following is most accurate? A. The agreement is likely partially integrated, and Butterfoss will be permitted to introduce evidence about the parties' agreed upon method for arranging a pick-up time. B. The parol evidence rule will not apply to the agreement since it is not integrated. C. Since the agreement is only partially integrated Holcomb will be permitted to introduce evidence about their discussions that Butterfoss promised to deliver the foosball table. D. The parol evidence will not bar evidence of extrinsic communications since the agreement is not for the sale of goods for over $500.

This is likely a partially integrated writing. Partially integrated writings are contracts that include some but not all of the terms to which the parties have agreed. Partially integrated writings are the final statement of the terms included in them. In other words, with respect to the terms written down, parol evidence will not be allowed to contradict those terms. But because partially integrated writings do not include all of the terms to which the parties agreed, parol evidence will be allowed to supplement the writing with consistent additional terms. The best answer is "A. The agreement is likely partially integrated, and Butterfoss will be permitted to introduce evidence about the parties' agreed upon method for arranging a pick-up time." The agreement is most likely partially (not fully) integrated. Evidence that is consistent and supplements the agreement is therefore permitted. Evidence of how to arrange the pick-up time is not inconsistent with the term that Holcomb will pick up the table, therefore it is likely admissible. Working backwards through the other answer choices, answer D is incorrect because the parol evidence rule does not depend on the purchase price of the goods (that's the statute of frauds). C is not the best answer because the agreement is not totally integrated, but it is integrated as to method of transfer--it specifies Holcomb will pick up. Evidence that Butterfoss will deliver is inconsistent with that term and will not be permitted. B is not the best answer because although the agreement is not totally integrated, it is at least partially integrated.

Henry sees his 10-year-old neighbor, Casey, playing outside on Friday afternoon. He walks over to her and says, "I'll pay you $30 if you actually mow my lawn tomorrow." Casey enthusiastically agrees to do so. Casey shows up on Saturday morning at 8am, ready to mow. Henry comes outside, however, and says, "Sorry, Casey, I've changed my mind. I'm going to mow it myself." Which of the following statements about this interaction is most likely accurate? A. Henry is in breach of contract. Casey accepted his offer on Friday. B. Henry has rejected Casey's offer to mow the lawn on Saturday morning. C. Henry has revoked his offer that he made to Casey on Friday. D. Henry cannot revoke the offer that he made to Casey on Friday because she has substantially performed.

This one is tough! C is the best option on the board. An offeror can revoke an offer at any time before acceptance (unless there's an option contract or an option-like mechanism holding the offer open). Here, Henry made an offer seeking performance as acceptance. He says that he'll pay Casey if she "actually mows" his lawn on Saturday. He doesn't want Casey's promise to perform but instead wants her actual performance. To accept such an offer, an offeree must perform the requested performance. Since Casey hasn't even begun mowing the law, Henry likely can revoke. B is tricky. From a certain perspective, it could be true, but it's not the best conceptualization of what's happening. Casey didn't show up on Saturday to make an offer. She showed up to mow the lawn and accept Henry's unilateral contract offer. So, Henry's response to Casey showing up isn't best understood as a rejection of Casey's offer. It's best understood as a revocation of Henry's offer. How about D? I'd say D is a tempting answer. To prevent offerors from taking advantage of offerees in unilateral contracts, an offeror may lose the ability to revoke once an offeree has rendered substantial or part performance (depending on the jurisdiction), giving the offeree a reasonable period of time to complete the requested performance and thus accept. (You can look at Restatement Section 45 to refresh your recollection. "Option Contract Created By Part Performance Or Tender -- (1) Where an offer invites an offeree to accept by rendering a performance and does not invite a promissory acceptance, an option contract is created when the offeree tenders or begins the invited performance or tenders a beginning of it.") The trouble with D is that it says Henry cannot revoke because Casey has provided "substantial" performance. She has not. She's not even started mowing. She has shown up to mow, which suggests that she's ready to perform, perhaps, but she's not substantially through the performance. If D had said something about part-performance, there'd be an argument. But Casey hasn't substantially performed. A isn't a great option. To be accurate, we'd have to conclude that Henry's offer was equivocal about whether he was making an offer for a unilateral or a bilateral contract. If there's doubt, the presumption is that an offeree can accept in any reasonable way. Casey does "enthusiastically agree" on Friday. But Henry's offer is pretty clear: he wants her to "actually mow" the lawn. He's not interested in her promise to mow. Given the context and the clarity of his language, this is very likely an offer for a unilateral contract.

NewCo hires Selva, a software engineer, to help the newly formed company built its website. Selva is to be paid $25,000 for the job, which is a reasonable sum for the work, "as soon as the website is up and running." Selva performs his work flawlessly, but for reasons unrelated to Selva's work, NewCo does not launch its website. Selva demands payment for his services, but NewCo, arguing that payment for those services was "conditional" upon the website launching, which did not happen. How would a court likely construe the agreement? A. NewCo will win because the contract used the term "as soon as" which creates an express condition. Since the condition is not met, NewCo's duty to pay Selva never ripens. B. NewCo will win because the website getting up and running is a constructive condition. C. Selva will win as long as NewCo drafted the contract, since ambiguities in a contract will be construed against the drafter. D. Selva will win because a court would likely construe the agreement as requiring payment by NewCo within a reasonable time after Selva had completed the promised services.

This question is based on R2K Section 227, Illustration 2. The best answer is D. The court is likely to construe the agreement as requiring payment by NewCo within a reasonable time after Selva had completed the promised services for a couple reasons. First, the language is not a clear expression of an express condition (e.g., the agreement does not use the term "express condition" or similar language) and construing the agreement as creating an express condition would subject Selva to forfeiture for services fully performed as agreed. In addition, the failure to launch the website was beyond Selva's control, and there is no indication that Selva assumed the risk of the failure. A is not the best answer because the language "as soon as" does not create an express condition. There is not necessarily magic language to create an express condition, though some terms, e.g,. "expressly conditioned" make a court more likely to find an express condition. B is not the best answer because it is premised on a misunderstanding of the term "constructive condition." Constructive conditions are implied by law when there is neither an express condition nor one implied by fact. Usually, constructive conditions applies when the parties have set out the duties they have to perform but do not specify the sequence of performance. Our textbook uses the example of the fancy restaurant meal -- the parties impliedly agree that the diners do not have to pay until after their meal has been consumed. Here, the parties did specify the order of performance -- NewCo did not owe Selva the payment until he completed his work. C is tempting, but is not as good as D. It's true that there is a rule of construction that courts interpret ambiguous terms against the drafter, but that is not likely why the court will construe the agreement as containing no express condition. A court is likely to construe the contract as not containing an express condition for the reasons explained above.

A liquor producing company, Du Nord Spirits intended to expand its manufacturing capacity. Du Nord purchased an existing distillery, and decided to renovate the whole distillery to prepare it for use. Du Nord contacted a producer of manufacturing equipment to rent the large amount of equipment it would need to renovate. The two parties agreed orally that the producer would deliver the equipment in six months and the company would be finished using the equipment four months later. However, after 14 months, the producer did not deliver the equipment, claiming that the contract falls under the Statute of Frauds. Which of the following is correct? A. The contract is within the Statute of Frauds, because the contract was not completed in one year. B. The contract is within the Statute of Frauds because of the large amount of equipment ordered. C. The contract is not within the Statute of Frauds, because the contract was for less than one year and does not fall under any other category of contract subject to the Statute of Frauds. D. The contract does not fall within the Statute of Frauds because it does not specify the amount of equipment ordered and does not fall within any other category of contract subject to the Statute of Frauds.

This question tests your knowledge of the "within one year" aspect of the Statute of Frauds. R2K Section 130 provides in relevant part that "(1) Where any promise in a contract cannot be fully performed within a year from the time the contract is made, all promises in the contract are within the Statute of Frauds until one party to the contract completes his performance." Pro tip: Carefully read the illustrations in R2K130. The best answer is: C. The Statute of Frauds applies to certain categories of contracts, such as contracts for the sale of real estate or contracts lasting more than one year. The year at issue under the one-year provision is measured from the date of making the contract rather than the date of the beginning of performance. Here, the contract does not come within the Statute of Frauds, because it was supposed to be completed within ten months from formation of the contract. Because the prospect of performance is measured from the point of making the contract (when completion within a year was possible) rather than from the time of the dispute (when completion within the first year is no longer possible), no signed writing is required. Hence, the contract does not fall within the Statute. A is incorrect because it is irrelevant whether the actual contract is completed within one year or not. A contract will fall under the Statute of Frauds, if at formation of the contract, completion of the contract is impossible within one year. In this case, completion was possible within one year and was planned for ten months. Thus, the contract does not fall under the Statute. B is also incorrect. The Statute of Frauds applies to certain categories of contracts, such as contracts for the sale of real estate, contracts for the sale of goods of $500 or more, or contracts lasting more than one year. It might be tempting to read this fact pattern incorrectly and mistake the rented equipment for a contract for the sale of goods. This fact pattern required the manufacturer to deliver rented equipment, and the amount of that equipment is not an issue. Therefore, this answer is incorrect. D is also incorrect. The Statute of Frauds applies to certain categories of contracts, such as contracts for the sale of real estate, contracts for the sale of goods of $500 or more, or contracts lasting more than one year. This contract is to rent manufacturing equipment and may be performed in a year or less, so the Statute of Frauds is not in issue. Therefore, this answer is incorrect.

Which of the following bargains is LEAST LIKELY to be declared unconscionable? A. One of a series of loans secured by a home, made at interest rates exceeding 30%, to borrowers with few assets, little education, and low income. B. The President & CEO of a large business signs a loan agreement, which conspicuously discloses an above average 20% interest rate and an arbitration clause. C. A high-priced installment appliance sale negotiated in Spanish with a written contract in English, after Seller made oral assurances to Buyer not contained in the written agreement. D. Rent-to-own retail installment sales contract with a "cross-collateral" clause that entitles Seller to repossess any and all items purchased if Buyer fails to pay on any one item purchased.

We discussed two types of unconscionability: procedural and substantive. Procedural unconscionability, referred to as "bargaining naughtiness" (p. 289) has to do with the process of contracting. A contract might be procedurally unconscionable if an important term is hidden in small type and buried deep in the contract. Contracts must also meet substantive conscionability standards. Our textbook uses the example of a contract that purports to require a person to give up their firstborn child as one that is substantively unconscionable. (p. 289) The best answer is "B. The President & CEO of a large business signs a loan agreement, which conspicuously discloses an above average 20% interest rate and an arbitration clause." There is not a good argument for procedural unconscionability here, because the above average interest rate and the arbitration clause were conspicuously disclosed. There is likely no bargaining power disparity, since the problem suggests one party is a president and CEO of a large business. The interest rate, standing alone, is not enough to amount to substantive unconscionability. Answer choice A, based on Gulfco of Louisiana v. Brantley (p. 289) is an example of a contract that a court might find unconscionable based on the totality of the circumstances: high interest rate, bargaining power differential between the parties, the repeated nature of the transaction (a series of loans), and the fact that the loan is secured by a home. Answer choice C, based on the Frostifresh Corp. case (p. 300) is not the best answer because negotiating a contract in one language but presenting a contract in another is likely to raise an issue of procedural unconscionability. Answer choice D, based on the Williams v. Walker-Thomas Furniture Co. case (p. 296) is a classic example of a contract that is vulnerable to a substantive unconscionability defense.

Butterfoss enters into an oral contract with Holcomb for Holcomb to grade Butterfoss's exams for $3,900. They agree that Holcomb will finish up the grading in time for submission to the Registrar, and Holcomb will grade in a "professional" manner. A week later, Butterfoss calls Holcomb to ask about the grading. Holcomb says that she has not started work on the project and she would like to be paid half the money up front. Butterfoss refuses. He shouts, "Deal's off!" and Butterfoss hires another professor to do the grading. He has to pay the other professor $5,000. Which of the following is most correct? A. There was no contract between Butterfoss and Holcomb. B. There is a contract between Butterfoss and Holcomb, and no one has breached. C. There is a contract between Butterfoss and Holcomb, and Butterfoss has breached. D. There is a contract between Butterfoss and Holcomb, and Holcomb has breached.

This tests the very important "who breached first" question. In this fact pattern, Prof. Butterfoss's statement that the "deal's off" and his action in hiring another professor amounts to a total breach. If Holcomb's inquiry about being paid up front was a total breach, then Butterfoss was justified in his actions. However, a mere inquiry about payment terms is not a material breach. Holcomb's inquiry is also not an anticipatory repudiation because it is not an unequivocal and definite statement that Holcomb will not perform. In this hypo, therefore, Butterfoss breached first, so the best answer choice is "C. There is a contract between Butterfoss and Holcomb and Butterfoss has breached." To push yourself a little bit further, what would Holcomb's damages be?

Law school relies on adjunct professors to teach one of the law school's critical programs. If the program is not successfully taught each year, the law school's accreditation is at risk. If the law school loses its accreditation, the school's students can no longer access federal loans, and the school will almost certainly go out of business. In spring semester, several adjunct professors were unable to finish teaching for various reasons. The law school contacts Experienced Professor, a recently retired professor who is one of the few teachers with the experience and skill to teach out the semester. Experienced Professor is willing to take on the job but demands a premium because she will have to cancel her planned vacation and she was just getting used to retirement. Experienced Professor offers to teach out the class at a cost to the law school of $25,000, which is several thousand dollars higher than the adjunct rate of $7500, but lower than a full-time professor rate. Law school objects to the price, and Experienced Professor says, "Take it or leave it." Law school accepts Experienced Professor's offer because law school cannot find another professor who is competent and willing to teach out the class immediately. Experienced Professor teaches out the class, and she submits a bill for $25,000. If law school later tries to avoid paying the higher fee, will law school be successful? A. No, because Experienced Professor performed the services required under the contract. B. Yes, because the contract was unconscionable. C. Yes, because law school had no reasonable alternative. D. Yes, because Experienced Professor's improper threat resulted in duress.

Under these facts, the defenses of duress and unconscionability are not available to the law school. Choices C & D are "duress-like" answers. Here, duress is not an effective defense. Duress requires "a party's manifestation of assent [be] induced by an improper threat by the other party that leaves the victim no reasonable alternative." R2K § 175(1). If so, "the contract is voidable by the victim." Id. Choice D is not the best answer because the facts do not support a finding that Professor has engaged in an improper threat—one of the basic elements of duress. See R2K § 176. Professor is demanding a premium because she will need to cancel her vacation. Professor is an independent contractor and can charge what she likes for her services. These actions are not threats to commit a tort or a crime or any of the other types of improper threats listed by the R2K § 176(1). Additionally, Professor does not owe any duty to the law school, so there is no bad faith being exerted. Other types of improper threats under Restatement § 176(2) —a use of power for illegitimate ends—is not suggested by the facts. This is more a case where Professor has some leverage because of market forces—i.e., the unavailability of other professors. Also, it cannot be said that the exchange is unfair. Professor is asking for a premium on her time because she was planning to go on vacation. This is analogous to a worker getting overtime pay of time and a half. Therefore, Choice D is not the best choice. Answer choice C is also not the best choice. The Law School may go out of business if it does not hire Professor but the lack of alternatives alone does not constitute duress. There also must be an improper threat. As noted above, that element is missing; therefore, this choice is not the best choice. Choice B addresses unconscionability. A court may refuse to enforce a contract or term on a finding that it is procedurally and substantively unconscionable. Procedural unconscionability may be demonstrated by (1) gross inequality in bargaining power, or (2) unfair surprise. Substantive unconscionability may be shown by: (1) overly harsh allocation of risks not justified by the circumstances, or (2) great price disparity. Choice B is not the best answer. Professor's "take it or leave it" offer alone does not make the contract unconscionable. Law School knows or should know that it relies on a program for which there are limited people with the skills to complete the work. Consequently, it should come as no surprise that hiring such a professor may be expensive. Furthermore, it is unlikely that the $25,000 charge would be considered substantively unconscionable. The charge is more than an adjunct professor would be paid, but still less than a full-time professor. However, Professor is giving up vacation. In essence, Professor is not charging a rate that shocks the conscience to the level needed to be substantively unconscionable. That leaves Choice A as the best answer. This contract is likely enforceable and the defenses of unconscionability and duress will not be effective.


Kaugnay na mga set ng pag-aaral

NURS 505 Exam 3 PrepU (25, 27) (39, 40, 41)

View Set

Chapter 02: Organization Strategy and Project Selection (1X2 & true/false)

View Set

Trauma, Crisis, Disaster, and related Disorders Assessment

View Set

Ch. 21: Respiratory Care Modalities

View Set

Chapter 18 The Circulatory System:Blood

View Set

Module 9 - The Firm & Market Structures

View Set